The second shifting theorem and the unit step function

Click For Summary
The discussion focuses on using the second shifting theorem to find the Laplace transform of a piecewise function defined as f(t) = t² for t < 4 and f(t) = t for t ≥ 4. The key transformation involves expressing f(t) in terms of the unit step function, resulting in f(t) = t²[1 - u(t-4)] + tu(t-4). Participants emphasize the need for clarity in applying the second shifting theorem, as the original question lacks detailed explanations. The solution process includes recognizing how to transition from t² to t at t = 4 by incorporating the unit step function. Overall, the discussion highlights the challenges of understanding the theorem's application in this context.
Rubik
Messages
95
Reaction score
0

Homework Statement



I am trying to do some revision for an upcoming exam and one question I am trying to figure out is

Use the second shifting theorem to find the Laplace transfrom of the following function:
f(t) = t2, t < 4
t, t ≥ 4

Homework Equations





The Attempt at a Solution


I just don't understand how to get from the question to
f(t) = t2[1 - u(t-4)] + tu(t-4)
I am really struggling with applying the second shifting theorem to express in terms of the unit step function I am failing to see how it works because nothing is explained in basic detail?
 
Physics news on Phys.org
Rubik said:

Homework Statement



I am trying to do some revision for an upcoming exam and one question I am trying to figure out is

Use the second shifting theorem to find the Laplace transfrom of the following function:
f(t) = t2, t < 4
t, t ≥ 4

Homework Equations





The Attempt at a Solution


I just don't understand how to get from the question to
f(t) = t2[1 - u(t-4)] + tu(t-4)
I am really struggling with applying the second shifting theorem to express in terms of the unit step function I am failing to see how it works because nothing is explained in basic detail?

Think of it this way. You start out with f(t) = t2. Then at t = 4 you want to take out the t2 and put in t, so you add the term u(t-4)(-t2+t).

Then put it all together:

f(t) = t2+u(t-4)(-t2+t)

Now, if you wish, you can collect terms on the various powers of t:

f(t) = t2(1-u(t-4)) + tu(t-4)
 
Question: A clock's minute hand has length 4 and its hour hand has length 3. What is the distance between the tips at the moment when it is increasing most rapidly?(Putnam Exam Question) Answer: Making assumption that both the hands moves at constant angular velocities, the answer is ## \sqrt{7} .## But don't you think this assumption is somewhat doubtful and wrong?

Similar threads

  • · Replies 6 ·
Replies
6
Views
2K
Replies
2
Views
953
  • · Replies 12 ·
Replies
12
Views
5K
  • · Replies 3 ·
Replies
3
Views
4K
Replies
8
Views
1K
  • · Replies 2 ·
Replies
2
Views
4K
  • · Replies 2 ·
Replies
2
Views
3K
  • · Replies 3 ·
Replies
3
Views
2K
  • · Replies 1 ·
Replies
1
Views
1K
  • · Replies 11 ·
Replies
11
Views
1K